Autor Tema: Ordinales

0 Usuarios y 1 Visitante están viendo este tema.

14 Noviembre, 2012, 11:45 am
Respuesta #10

Raúl Aparicio Bustillo

  • $$\Large \color{#9c57a6}\pi\,\pi\,\pi\,\pi\,\pi\,\pi$$
  • Mensajes: 3,113
  • Karma: +0/-3
  • Sexo: Masculino
Es verdad, hablaba todo el rato de la cota del modelo, cuando me refería a la cota de la sucesión de ordinales del modelo. Pero nuestro modelo tendrá una sucesión de ordinales. Por ejemplo, imaginemos que nuestro modelo internamente tiene el ordinal\(  \omega +1 , \omega +2,\omega+3 y \omega+n  \)para cualquier ordinal finito n, pero sin embargo, no contiene un ordinal que sea \omega+omega, entonces diríamos que \( \omega\cdot{2}  \)es el supremo ( internamente al modelo) de la serie de ordinales. No sé si ahora me estoy explicando mejor. ¿ Cual sería esa cota como mínimo, y como máximo, para los modelos de ZFC. Por ejemplo, un ordinal \( \lambda \), cuya existencia no se pueda probar en ZFC, pues entonces será una cota para esa sucesión transfinita de ordinales. Aunque vuelvo aquí a la cuestión eterna de como se comparan para decir iguales objetos que están en distintos modelos de una teoría

14 Noviembre, 2012, 12:19 pm
Respuesta #11

Carlos Ivorra

  • Administrador
  • Mensajes: 11,114
  • País: es
  • Karma: +0/-0
  • Sexo: Masculino
    • Página web personal
Pero es que no es tan sencillo. Por ejemplo, en ZFC puedes demostrar la existencia de \( \aleph_1 \), pero esto no significa que \( \aleph_1 \) deba estar en todo modelo de ZFC. Puedes tener un modelo transitivo numerable \( M \) tal que \( M\cap \Omega \) (lo que llamas el supremo de M) sea un ordinal numerable. Lo que sucede es que en M habrá un ordinal numerable que será \( \aleph_1 \) para quienes "vivan en M".

Por ejemplo, un ordinal \( \lambda \), cuya existencia no se pueda probar en ZFC, pues entonces será una cota para esa sucesión transfinita de ordinales.

¿Quieres decir una cota superior o inferior? Porque no es ni lo uno ni lo otro. No puedes demostrar en ZFC que existan cardinales inaccesibles, pero nada te impide tener un modelo M tal que \( M\cap \Omega \) sea menor que el mínimo cardinal inaccesible (si es que existe uno) o mayor que dicho mínimo cardinal inaccesible.

Si hablamos de modelos cualesquiera, no necesariamente transitivos, entonces puedes tener modelos tales que la clase de sus ordinales no esté bien ordenada externamente, con lo que ni siquiera puedes comparar sus ordinales con ordinales "de verdad". Si te restringes a modelos transitivos, entonces, suponiendo que exista un modelo de ZFC, puedes definir el mínimo ordinal \( \xi \) tal que existe un modelo transitivo \( M \) de ZFC tal que \( M\cap \Omega = \xi \), que coincide con el mínimo ordinal \( \xi \) tal que \( L_\xi \) es un modelo de ZFC, y entonces \( \xi \) es un ordinal numerable (si es que existe), pero será mayor que el menor ordinal no recursivo, lo cual implica que no puedes definir explícitamente un buen orden en \( \omega \) cuyo ordinal sea \( \xi \).

15 Noviembre, 2012, 02:45 pm
Respuesta #12

Raúl Aparicio Bustillo

  • $$\Large \color{#9c57a6}\pi\,\pi\,\pi\,\pi\,\pi\,\pi$$
  • Mensajes: 3,113
  • Karma: +0/-3
  • Sexo: Masculino
Citar
Lo que sucede es que en M habrá un ordinal numerable que será [\aleph_1] para quienes "vivan en M".

Me refiero a los ordinales vistos desde dentro del modelo. Obviamente si estamos en un modelo numerable \( \aleph_1 \) no va a estar exteriormente al modelo, pero si interiormente

15 Noviembre, 2012, 10:35 pm
Respuesta #13

Carlos Ivorra

  • Administrador
  • Mensajes: 11,114
  • País: es
  • Karma: +0/-0
  • Sexo: Masculino
    • Página web personal
Me refiero a los ordinales vistos desde dentro del modelo. Obviamente si estamos en un modelo numerable \( \aleph_1 \) no va a estar exteriormente al modelo, pero si interiormente

Vale, pero sigo diciéndote que tu pregunta está mal planteada, en el sentido de que no veo qué clase de respuesta podría tener. No digo que no sepa la respuesta, sino que no entiendo qué podría ser una respuesta.

16 Noviembre, 2012, 08:24 am
Respuesta #14

Raúl Aparicio Bustillo

  • $$\Large \color{#9c57a6}\pi\,\pi\,\pi\,\pi\,\pi\,\pi$$
  • Mensajes: 3,113
  • Karma: +0/-3
  • Sexo: Masculino
Pues a ver, el supremo ( o lo que yo pretendo llamar supremo), tiene que ser un ordinal límite. No puede ser un ordinal sucesor, porque si el supremo  fuera por ejemplo \( \omega+4 \), yo uso el teorema de la unión ( la unión de 2 conjuntos es un conjunto), cojo \( \omega+4\cup{\{omega+4\}} \), y ya es el siguiente, luego el anterior no podría ser una cota.

16 Noviembre, 2012, 07:35 pm
Respuesta #15

Carlos Ivorra

  • Administrador
  • Mensajes: 11,114
  • País: es
  • Karma: +0/-0
  • Sexo: Masculino
    • Página web personal
Pues a ver, el supremo ( o lo que yo pretendo llamar supremo), tiene que ser un ordinal límite.

Tiene que ser un ordinal límite supuesto que sea un ordinal, pues el conjunto de los objetos de un modelo que satisfacen la definición de ordinal no tiene por qué estar bien ordenado. Si el modelo es natural, entonces sí, y será isomorfo a un ordinal, que de hecho será un ordinal límite, pero tú preguntas cuál es el mínimo y el máximo valor posible para dicho ordinal límite, y lo que te digo es que puede ser arbitrariamente grande (no hay un máximo valor posible) y en cada modelo de ZFC en el cual haya un modelo transitivo de ZFC habrá un mínimo ordinal que sea el supremo de un modelo de ZFC, pero así tienes un ordinal en cada modelo, no sé en qué sentido se puede responder a la pregunta de cuál es el mínimo valor posible para ese ordinal.

17 Noviembre, 2012, 08:39 am
Respuesta #16

Raúl Aparicio Bustillo

  • $$\Large \color{#9c57a6}\pi\,\pi\,\pi\,\pi\,\pi\,\pi$$
  • Mensajes: 3,113
  • Karma: +0/-3
  • Sexo: Masculino
Pues cogemos los ordinales de todos los modelos ( más bien su notación, por si acaso el \omega3 de uno de los modelos es un ordinal distinto en el otro, o no es un ordinal) , y el primer valor ( o el equivalente por notación) que haya a esos ( pero siempre de cada ordinal desde su modelo), serán los supremos ( resp. ínfimos) de cada modelo

17 Noviembre, 2012, 03:54 pm
Respuesta #17

Carlos Ivorra

  • Administrador
  • Mensajes: 11,114
  • País: es
  • Karma: +0/-0
  • Sexo: Masculino
    • Página web personal
Pues cogemos los ordinales de todos los modelos ( más bien su notación, por si acaso el \omega3 de uno de los modelos es un ordinal distinto en el otro, o no es un ordinal) , y el primer valor ( o el equivalente por notación) que haya a esos ( pero siempre de cada ordinal desde su modelo), serán los supremos ( resp. ínfimos) de cada modelo

Pero es que esto no tiene sentido. Cuando dices "todos los modelos", ¿qué quieres decir? ¿Te refieres a todos los modelos que haya en un modelo dado de ZFC?

Cuando dices "su notación", ¿qué quieres decir?, porque es cierto que algunos ordinales tienen nombre, como "\omega3", pero ¿qué "notación" corresponde al mínimo ordinal \( \alpha \) tal que existe un modelo transitivo M de ZFC tal que \( M\cap \Omega=\alpha \)? (Teniendo en cuenta además que podría no haber ningún \( \alpha \) en esas condiciones.)

Cuando dices "el primer valor" ¿qué quieres decir? En cualquier modelo, "el nombre" del conjunto de todos los ordinales de ese modelo (puestos a darle un nombre) será \( \Omega \), la clase de todos los ordinales. Lo cual no quita para que ese \( \Omega \) en un modelo pueda ser un \( \lambda\in\Omega \) en otro modelo y que ese \( \lambda \) no tenga ningún "nombre" en particular.

17 Noviembre, 2012, 05:13 pm
Respuesta #18

Raúl Aparicio Bustillo

  • $$\Large \color{#9c57a6}\pi\,\pi\,\pi\,\pi\,\pi\,\pi$$
  • Mensajes: 3,113
  • Karma: +0/-3
  • Sexo: Masculino
Bueno, vamos a ver. En cualquier modelo infinito existe \( \omega \) ,¿no? Pues puedo asegurar que existe \( \omega+1 \), que no es más que\(  \omega\cup{\omega} \), y por recursión finita, que existe\(  \omega+n \), para cualquier n finito, ahora bien, quizás por otros axiomas sí, pero por este procedimiento no puedo demostrar que exista\(  \omega+\omega \), pero si que existe cualquier ordinal menor que él, luego podríamos decir que \(  \omega+omega \) es el supremo, o de no serlo, que existirá un supremo. No podemos decir que igual que en una sucesión infinita de los ordinales finitos, están todos los ordinales finitos, aquí están todos los ordinales, por el simple hecho de estar en ZFC. Por el simple hecho de estar en ZFC y el axioma de infinitud podemos afirmar que existe el ordinal \( \omega \), pero no podemos decir que existe un cardinal fuertemente inaccesible, de hecho, podría ser ZFC consistente e inconsitente ese cardinal fuertemente inaccesible. Al igual que los cardinales finitos tienen un supremo ( que no es finito) , tendrá que haber algun supremo para los cardinales infinitos, que podrá variar de unos modelos a otros, pero tendrá que ser definible, aunque será una clase propia. No sé si ahora me estoy explicando mejor

17 Noviembre, 2012, 09:02 pm
Respuesta #19

Carlos Ivorra

  • Administrador
  • Mensajes: 11,114
  • País: es
  • Karma: +0/-0
  • Sexo: Masculino
    • Página web personal
Pues no sé qué decirte. Sigo sin verle el sentido a tu pregunta. El supremo de los cardinales infinitos es la clase \( \Omega \) de todos los ordinales. Lo cual no quita para que en unos modelos esa clase pueda ser más grande o menos grande que en otros. Pero en todos ellos será \( \Omega \). En el supuesto de que puedas comparar los ordinales de dos modelos, por ejemplo si ambos son modelos transitivos de ZFC en un mismo modelo de ZFC, entonces lo que en uno de los modelos es \( \Omega \) en otro podrá ser un ordinal límite \( \lambda<\Omega \), pero, si en un modelo de ZFC existen modelos transitivos de ZFC, los hay con un conjunto de ordinales tan grande como se quiera, si pretendes hablar del supremo de todos los supremos posibles te vuelves a encontrar con el \( \Omega \) del modelo grande, y si quieres hablar del menor supremo posible, te encuentras con eso, con el menor ordinal \( \lambda \) tal que existe un modelo transitivo M de ZFC con \( M\cap \Omega =\lambda \), pero ese ordinal \( \lambda \) será un cierto ordinal numerable que no vas a poder expresar en términos de la aritmética ordinal a partir de \( \omega \). Por eso te digo que no veo sentido a que preguntes cuál es. Es el que es. No es que no te sepa decir cuál es, sino que no sé qué podría ser una respuesta a tu pregunta. Pareces creer que todo ordinal numerable se puede definir en términos de \( \omega \), pero eso no es así.